felix termodinamica quimica ch03

Embed Size (px)

Citation preview

  • 8/13/2019 felix termodinamica quimica ch03

    1/104

    Chapter 3 Solutions Engineering and Chemical Thermodynamics

    Wyatt TenhaeffMilo Koretsky

    Department of Chemical EngineeringOregon State University

    [email protected]

  • 8/13/2019 felix termodinamica quimica ch03

    2/104

    2

    3.1Since entropy is a state function

    2,1, stepsysstepsyssys sss +=

    Step 1 is a constant volume process. Therefore, no work is done. After neglecting potential ankinetic energy effects, the energy balance for a reversible process becomes

    qu = qdu = qdT cv =

    Using the definition of entropy,

    ==

    final

    initial

    T

    T

    vrevstepsys

    T

    dT c

    T

    qs

    2

    1

    1,

    Step 2 is an isothermal process. For and ideal gasu is zero and the energy balance is (PE andKE neglected)

    wq = wdq =

    For an ideal gas, the following can be shown

    dvv RT

    Pdvwrev ==

    Therefore,

    ===

    1

    22, ln

    2

    1vv

    Rdvv

    RT q

    s final

    initial

    v

    v

    revstepsys

    Combination of both steps yields

    += 1

    2ln2

    1vv R

    T dT cs

    T

    T

    vsys

  • 8/13/2019 felix termodinamica quimica ch03

    3/104

    3

    3.2

    Equation 3.62 states

    =

    2

    112

    ln

    T

    T

    Psys P

    P RdT T

    cs

    Substituting the equation forPc yields

    ++=

    2

    112

    2ln

    T

    T sys P

    P RdT

    T CT BT A

    s

    ( ) ( )

    ++

    =

    122

    12212

    12 ln

    2ln

    PP

    RT T C

    T T BT T

    Assys

  • 8/13/2019 felix termodinamica quimica ch03

    4/104

    4

    3.3Equation 3.3 states

    surr sysuniv S S S +=

    where

    )( 12 ssmS sys =

    surr

    surr surr T

    QS = (the temperature of the surroundings is constant)

    First, we will calculate sysS . From the steam tables:

    =K kg

    kJ 1228.71s (300 C, 10 bar)

    Since the container is rigid and mass is conserved in the process,

    12 vv =

    =kgm 25794.0

    31v (300 C, 10 bar)

    =kgm 25794.0

    32v

    To find the number of phases present in state 2, compare the specific volume of state 2 to thespecific volume for saturated water and saturated water vapor at 1 bar. From the steam tables,

    =kgm 001043.0

    3,

    2sat lv (sat. H2O(l) at 1 bar)

    =kgm 6940.1

    3,

    2sat vv (sat. H2O(v) at 1 bar)

    Since sat vsat l vvv ,22,

    2

  • 8/13/2019 felix termodinamica quimica ch03

    5/104

    5

    152.0= x

    From the steam tables:

    = K kgkJ 3025.1,2 sat ls (sat. H2O(l) at 1 bar)

    =K kg

    kJ 3593.7 ,2sat vs (sat. H2O(v) at 1 bar)

    The entropy of state 2 can be calculated using the following equation:

    ( ) sat vsat l s xs xs ,2,

    22 1 +=

    =K kg

    kJ 2231.22

    s

    Now the entropy change of the system can be calculated.

    [ ]( ) =

    =K kJ 0.49

    K kgkJ 1228.7

    K kgkJ 2231.2kg10sysS

    To find the change in entropy of the surroundings, an energy balance will be useful. Since nowork is done, the energy balance is

    QU = We also know

    QQsurr =

    To following expression is used to solve for the internal energy:

    ( )( 1,2,2 1 u xuu xmU sat vsat l +=

    From the steam tables

    =kgkJ 2.27931u (300 C, 10 bar)

    =kgkJ 33.417 ,2

    sat lu (sat. H2O(l) at 1 bar)

  • 8/13/2019 felix termodinamica quimica ch03

    6/104

    6

    =kgkJ 1.2506 ,2

    sat vu (sat. H2O(v) at 1 bar)

    The expression for internal energy yields

    [ ]kJ77.20583=U

    Therefore,

    [ ]kJ77.20583=Q

    and

    [ ][ ]

    ===K kJ 25.70

    K 293kJ77.20583

    surr

    surr surr T

    QS

    Now the change in entropy of the universe can be calculated

    =+=+=K kJ 25.21

    K kJ 25.70

    K kJ 0.49surr sysuniv S S S

    SinceS univ > 0, this process is possible.

  • 8/13/2019 felix termodinamica quimica ch03

    7/104

    7

    3.4Entropy Balance:

    surr sysuniv S S S +=

    Considering the copper block to be the system, no work is done on the system; thus, the energy balance is

    qdu = (neglecting PE and KE)qdT cv =

    This can be used in the following expression for entropy

    =2

    1

    T

    T

    revsys T

    qs

    =2

    1

    T

    T

    vsys dT T

    cs

    From Table A.2.3

    RcP 723.2= Rcc Pv 723.2== (for liquids and solids)

    Therefore,

    [ ][ ]

    =

    == K 15.373

    K 280lnK mol

    J 314.8723.2ln723.2723.212

    2

    1T T

    RdT T

    Rs

    T

    T sys

    =K mol

    J 50.6syss

    [ ] =

    =K J 1024

    K molJ 50.6

    molkg

    063465.0

    kg10sysS

    Since the temperature of the lake remains constant, the change in entropy of the surroundings c be calculated as follows

    ( ) ( )surr

    Cu

    surr

    PCu

    surr surr

    surr surr T

    T T RnT

    T T cnT

    QT Q

    S 1212723.2 ====

  • 8/13/2019 felix termodinamica quimica ch03

    8/104

    8

    [ ] [ ] [ ]( )

    [ ]

    =K 280

    K 15.373K 280K mol

    J 314.8723.2

    molkg 0.063546

    kg10surr S

    =K J 1184surr S

    From the definition of entropy:

    =+=+=K J 160

    K J 1184

    K J 1024surr sysuniv S S S

  • 8/13/2019 felix termodinamica quimica ch03

    9/104

    9

    3.5

    (a)Since the process is adiabatic and reversible,

    0= sysS

    (b)The change in entropy is calculated by

    ( )=

    === K J 9.73

    K 111molkJ2.8mol1liquid

    vapor b

    vaprev

    sys T hn

    T q

    nS

    The sign is negative because there is less randomness in the liquid phase.

    (c)For this situation

    dT cq P=

    Therefore,

    ( )

    === K 15.373K 15.273ln

    K gJ2.4g0148.18

    K 15.273

    K 373.15

    f

    i

    T

    T

    Prevsys dT T

    cT q

    mS

    =K J6.23sysS

    The sign is negative because as the water cools, less translational energy states are occupied bythe molecules. Therefore, the randomness decreases.

    (d)First, calculate the temperature at which the blocks (block A and block B) equilibrate. Theenergy balance for the process is

    ( ) ( ) 0K 15.473K 15.373 22 =+ T cnT cn P BP A Since the heat capacities number of moles of A and B are equal, we find that

    K 15.4232 =T

    Now, calculate the change in entropy:

  • 8/13/2019 felix termodinamica quimica ch03

    10/104

    10

    Bsys Asyssys S S S ,, +=

    +

    =

    BP B

    AP Asys T

    T cn

    T T

    cnS ,12

    ,12 lnln

    Substituting values, we obtain

    =K J 337.0sysS

    The sign is positive because two objects at different temperatures will spontaneously equilibratto the same temperature when placed together.

  • 8/13/2019 felix termodinamica quimica ch03

    11/104

    11

    3.6

    The solution below compares problems 2.14 and 2.15, the calculation of 2.13 was erroneouslyincluded in the problem statement of the first printing and is shown at the end of this problem.

    Problem 2.14Since the system is well-insulated no heat is transferred with the surroundings. Therefore, theentropy change of the surroundings is zero and

    sysuniv S S =

    The gas in the piston-cylinder system is ideal andPc is constant, so

    =

    12

    12 lnln

    PP

    RT T

    cnS Psys

    From the problem statement, we know

    [ ][ ] bar 1 bar 2

    2

    1==

    P

    P

    The ideal gas law can be used to solve1T .

    [ ]( )( )

    [ ]( )[ ]K 6.240

    mol1K mol

    bar L 0.08314

    L10 bar 2111 =

    ==

    nRV P

    T

    Solving for 2T is slightly more involved. The energy balance for this system where potential andkinetic energy effects are neglected is

    W U =

    Conservation of mass requires

    21 nn = Let

    21 nnn ==

    The energy balance can be rewritten as

    =2

    1

    2

    1

    V

    V E

    T

    T v dV PdT cn

  • 8/13/2019 felix termodinamica quimica ch03

    12/104

    12

    Since vc and E P are constant

    ( ) ( )1212 V V PT T nc E v =

    2V and 1T can be rewritten using the ideal gas law

    22

    2 PnRT

    V =

    nRV P

    T 111 =

    Substituting these expressions into the energy balance, realizing that 2PP E = , and simplifyingthe equation gives

    nR

    V PPT

    27

    25

    1122

    +

    =

    Using the following values

    [ ][ ]

    [ ][ ]

    =

    ==

    ==

    K mol bar L 08314.0

    mol0.1 L10

    bar 1 bar 2

    1

    2

    1

    R

    nV

    P

    P

    results in

    [ ]K 2062 =T

    Since both states are constrained, the entropy can be calculated from Equation 3.63:

    [ ]( ) [ ][ ][ ][ ]

    ==

    bar 2 bar 1ln

    K molJ 314.8

    K 6.240K 206ln

    K molJ 314.85.3mol0.1sysuniv S S

    ==K J 24.1sysuniv S S

  • 8/13/2019 felix termodinamica quimica ch03

    13/104

    13

    Problem 2.15Since the initial conditions in the piston-cylinder assembly are equal to the initial conditions ofProblem 2.14, 1T and 1P are known. Moreover,2P is known, so we only need to find2T inorder to calculate the entropy change. For adiabatic, reversible processes, the followingrelationship (Equation 2.89) holds:

    const PV k =

    This can be used to find2V .

    k k V PP

    V

    1

    121

    2

    =

    Noting that5

    7==v

    P

    c

    ck and substituting the proper values results in

    [ ]L4.162 =V

    Th polytropic expression can also be manipulated to yield

    .1 const TV k =

    Therefore,

    12

    1112

    =k

    k

    V

    V T T

    Substitution of the appropriate variables provides

    [ ]K 4.1972 =T

    Now the entropy can be calculated,

    [ ]( ) [ ]

    [ ][ ][ ]

    == bar 2 bar 1

    lnK molJ

    314.8K 6.240K 97.41

    lnK molJ

    314.85.3mol0.1sysuniv S S

    ==K J 004.0sysuniv S S

    The value of the entropy shown represents round-off error. Since the process is reversible andadiabatic, we know from Table 3.1 and the related discussion in Section 3.3 that the entropychanges of the system, surroundings, and universe will be zero.

  • 8/13/2019 felix termodinamica quimica ch03

    14/104

    14

    Problem 2.13

    Equation 3.3 states

    surr

    surr surr sysuniv

    T

    QssmS S S +=+= )( 12

    where the temperature of the surroundings is constant. First, we will determine2s . The first lawcan be applied to constrain state 2. With potential and kinetic energy effects neglected, theenergy balance becomes

    W QU +=

    The value of the work will be used to obtain the final temperature. The definition of work(Equation 2.7) is

    =2

    1

    V

    V E dV PW

    Since the piston expands at constant pressure, the above relationship becomes

    ( )12 V V PW E =

    From the steam tables

    =K kg

    kJ 2119.61s (10 MPa, 400 C)

    =kgm 02641.0

    31v (10 MPa, 400 C)

    [ ]33111 m07923.0kgm 02641.0kg)3( =

    == vmV

    Now 2V and 2v are found as follows

    [ ]36312 m4536.0Pa100.2J748740m07923.0 =

    ==

    E PW

    V V

    [ ] ===

    kgm 1512.0

    kg3m4536.0

    33

    22

    2 mV

    v

  • 8/13/2019 felix termodinamica quimica ch03

    15/104

    15

    Since 2v and 2P are known, state 2 is constrained. From the steam tables:

    =K kg

    kJ 1270.72s

    kg

    m 0.1512 bar,203

    Now U will be evaluated, which is necessary for calculatingsurr Q . From the steam tables:

    =kgkJ 2.29452u

    kgm 0.1512 bar,20

    3

    =kgkJ 4.28321u ( )C400 bar,001

    ( ) [ ]( ) [ ]kJ4.338kgkJ 4.2832

    kgkJ 2.2945kg3 121 =

    == uumU

    Substituting the values ofU and W into the energy equation allows calculation of Q

    W U Q = [ ]( ) [ ] surr QQ === J1009.1J748740[J]338400 6

    so

    [ ]( ) [ ][ ] =

    =

    +=

    K kJ 126.1

    K 15.673kJ1009.1

    K kgkJ 2119.6

    K kgkJ .12707kg3

    )(

    3

    12surr

    surr univ T

    QssmS

    Therefore, this process is irreversible.

  • 8/13/2019 felix termodinamica quimica ch03

    16/104

    16

    3.7(a)

    From Equation 3.63:

    =

    1

    2

    1

    2 lnlnPP R

    T T cs Psys

    [ ][ ]

    [ ][ ]

    =

    =

    K molJ 63.20

    bar 1 bar 5.0ln

    K 300K 500ln

    27

    K molJ 314.8syss

    (b)From Equation 3.65:

    +

    =1

    2

    1

    2 lnlnv

    v R

    T

    T cs vsys

    [ ][ ]

    =

    +

    =

    K molJ 85.4

    molm 05.0

    molm .0250

    lnK 300K 500ln

    25

    K molJ 314.8

    3

    3

    syss

    (c)First, we can find the molar volume of state 1 using the ideal gas law.

    == molm 025.03

    111 P RT v

    Now, we can use Equation 3.65

    +

    =

    1

    2

    1

    2 lnlnvv

    RT T

    cs vsys

    [ ][ ]

    =

    +

    =K mol

    J 62.10

    molm 025.0

    molm .0250

    lnK 300

    K 500ln2

    5

    K mol

    J 314.83

    3

    syss

  • 8/13/2019 felix termodinamica quimica ch03

    17/104

    17

    3.8(i). We wish to use the steam tables to calculate the entropy change of liquid water as it goesfrom its freezing point to its boiling point. The steam tables in Appendices B.1 B.5 do nothave data for subcooled water at 1 atm. However, there is data for saturated water at 0.01 C ana pressure of 0.6113 kPa. If we believe that the entropy of water is weakly affected by pressure

    then we can say that the entropy of water at 0.01 C and 0.6113 kPa is approximately equal to thentropy at 0 C and 1 atm. The molar volumes of most liquids do not change much with pressuat constant temperature. Thus, the molecular configurations over space available to the watermolecules do not change, and the entropy essentially remains constant. We do not need toconsider the molecular configurations over energy since the temperature difference is so slight.So, from the steam tables:

    ( ) ( ) = K kgkJ 0kPa.61130,01.0atm1,0 C sC s

    ( ) =K kg

    kJ 3068.1atm1,100 C s

    Therefore,

    ( ) ( ) == K kgkJ 0

    K kgkJ 3068.1atm1,0atm1,100 C sC ss

    =K kg

    kJ 3068.1s

    (ii). From the steam tables:

    ( )=K kg

    kJ 3068.1atm1,100 C s l

    ( )=K kg

    kJ 3548.7atm1,100 C s v

    Therefore,

    ( ) ( ) == K kgkJ 3068.1

    K kgkJ 3548.7atm1,100atm1,100 C sC ss lv

    =K kg

    kJ 048.6s

    The change in entropy for process (ii) is 4.63 times the change in entropy for process (i). Thereare many ways to reconcile this difference, but think about it from a molecular point of view. I process (i), the available molecular configurations over energy are increased as the temperatureincreases. As the temperature increases, the molar volume also increases slightly, so the

  • 8/13/2019 felix termodinamica quimica ch03

    18/104

    18

    available molecular configurations over space also increase. Now consider process (ii), wherethe molecules are being vaporized and entering the vapor phase. The molecular configurationover space contribution to entropy is drastically increased in this process. In the liquid state, thmolecules are linked to each other through intermolecular interactions and their motion islimited. In the vapor state, the molecules can move freely. Refer to Section 3.10 for a discussio

    of entropy from a molecular view.

  • 8/13/2019 felix termodinamica quimica ch03

    19/104

    19

    3.9Before calculating the change in entropy, we need to determine the final state of the system. Lethe mixture of ice and water immediately after the ice has been added represent the system.Since the glass is adiabatic, no work is performed, and the potential and kinetic energies areneglected, the energy balance reduces

    0= H

    We can split the system into two subsystems: the ice (subscripti) and the water (subscriptw).Therefore,

    0=+= wwii hmhm H and

    wwii hmhm =

    We can get the moles of water and ice.

    [ ] [ ]kg399.0kgm 001003.0

    m.00040 3

    3

    =

    ==

    w

    ww v

    V m

    ( )[ ]( ) [ ]mol15.22

    molkg 0180148.0

    kg399.0

    2

    ===O H

    ww MW

    mn

    ( ) [ ]mol55.5

    2

    ==O H

    i

    i MW

    mn

    Now, lets assume that all of the ice melts in the process. (If the final answer is greater than 0C, the assumption is correct.) The following expression mathematically represents the changein internal energies (cP=c v).

    ( )( ) C250C100 ,,, =+ f wPw f wP fusiPi T cnC T chcn Note: Assumed the heat capacities are independent of temperature to obtain thisexpression.

    From Appendix A.2.3 Rc iP 196.4, = Rc wP 069.9, =

    and

    =molkJ0.6 fush

  • 8/13/2019 felix termodinamica quimica ch03

    20/104

    20

    Substitution of values into the above energy balance allows calculation ofT f .

    C 12.3= f T

    (Our assumption that all the ice melts is correct.)

    Now, we can calculate the change in entropy. From Equation 3.3

    surr sysuniv sss +=

    Since the glass is considered adiabatic,

    0= surr s sysuniv ss =

    We will again break the system into two subsystems: the ice and the water. The change inentropy of the universe can be calculated as follows

    wwiiuniv snsnS +=

    The definition of change in entropy is

    = final

    initial

    rev

    T q

    s

    Assuming the heat capacities of ice and water are independent of temperature, the expressionsfor the change in entropy of the subsystems are

    +

    +

    =

    K 15.273K 27.276ln

    K 15.273K 15.263K 15.273ln ,, wP

    fusiPi c

    hcs

    =

    K 98.152K 27.276ln,wPw cs

    Therefore

    +

    +

    +

    =

    K 98.152K 27.276ln

    K 15.273K 27.276ln

    K 15.273K 15.263K 15.273ln ,,, wPwwP

    fusiPiuniv cnc

    hcnS

    Substituting the values used before, we obtain

  • 8/13/2019 felix termodinamica quimica ch03

    21/104

    21

    =mol

    J 59.6univS

  • 8/13/2019 felix termodinamica quimica ch03

    22/104

    22

    3.10

    (a)The maximum amount of work is obtained in a reversible process. We also know the entropychange for the universe is zero for reversible processes. From the steam tables

    =K kg

    kJ 2119.61s (400 C, 100 bar)

    =K kg

    kJ 5434.82s (400 C, 1 bar)

    Using the entropy criterion,

    ( ) surr univ S ssmS +== 12 0

    ( ) [ ]( )

    == K kgkJ

    2119.6K kgkJ

    5434.8kg5.0 12 ssmS surr

    =K kJ 1658.1surr S

    Since the process is isothermal, we know that the temperature of the surroundings is constant at400 C. Therefore,

    ==K kJ 1658.1surr

    surr

    surr S T Q

    [ ]kJ76.784=surr Q

    To find the work obtained, perform an energy balance. An energy balance where potential andkinetic energy effects are neglected is

    W QU +=

    We can calculate the change in internal energy from the steam tables, and we also know thatsurr QQ = . From the steam tables:

    = kgkJ 4.28321u (400 C, 100 bar)

    =kgkJ 8.29672u (400 C, 1 bar)

    Therefore,

  • 8/13/2019 felix termodinamica quimica ch03

    23/104

  • 8/13/2019 felix termodinamica quimica ch03

    24/104

  • 8/13/2019 felix termodinamica quimica ch03

    25/104

    25

    (d)Since the system is well-insulated

    0= surr s

    Use Equation 3.65 to calculate the change in entropy:

    ( )

    +

    =

    +

    ==

    2112

    12

    12

    12 lnln2/5lnln

    PT PT

    RT T

    RV V

    RT T

    css vsysuniv

    Substituting values, we obtain,

    =K mol

    J 354.0syss

    (e)Because the change in entropy of the universe is equal to change in entropy of the system, whicis positive in this situation, the second law is not violated.

  • 8/13/2019 felix termodinamica quimica ch03

    26/104

    26

    3.12

    (a)To calculatessys, we need to take the gas from state 1 to state 2 using a reversible process. The process in part a can be drawn as follows:

    State 1

    24.6 bar500 K

    State 2

    34.4 bar557 K

    State I

    P I T I = 557 K

    rev.adiabatic rev.

    isothermal

    We need to find the intermediate pressure,P I where we end up at the temperature in state 2,T 2.To find it, we can use the results from pages 78-79 of the text:

    ( ) ( ) ( ) k k I

    k k I I

    k k PT PT PT /12/1/1

    11 ==

    Therefore,

    ( )

    [bar]9.3511

    2

    1

    =

    =

    PT T

    P

    k k

    I (I)

    If we draw the process on aPT diagram, we get:

    20

    40

    490 560

    1

    2

    T

    P

    I

    Adiabatic compression Isothermalexpansion

  • 8/13/2019 felix termodinamica quimica ch03

    27/104

    27

    The change in entropy can be represented as follows:

    isothermaladiabaticsys sss +=

    For the reversible adiabatic process, the change in entropy is zero. Therefore,

    isothermalsys ss =

    For an isothermal process,

    dPP

    RT wq revrev ==

    Therefore,

    =

    === K molJ 354.0ln

    22

    I

    P

    P

    final

    initialsys P

    P RdPP

    RT qs

    I

    If we substitute relation (I) in the expression above, we get the same expression in the book:

    ( )

    =

    +

    =

    1

    2

    1

    2

    1

    2

    1

    2 lnlnln1

    lnPP

    RT T

    cT T

    Rk

    k PP

    Rs Psys

    (b)For this construction, we use the path diagrammed on the followingPT diagram:

    20

    40

    490 560

    1

    2

    T

    P

    Isobaric heating

    Isothermalcompression

    We write

  • 8/13/2019 felix termodinamica quimica ch03

    28/104

    28

    isothermalisobar sys sss +=

    First, find an expression for the isobaric heating

    dT cdhq Prev ==

    Therefore,

    ==

    12ln

    2

    1T T

    cdT T c

    s P

    T

    T

    Pisobar

    Now, we need an expression for the isothermal, reversible expansion

    Pdvwq revrev == Therefore,

    =

    ===

    =

    =

    =

    = 12

    /

    / 21

    /

    /lnln

    222

    121

    222

    12PP

    RPP

    Rdvv

    Rdv

    T P

    sP RT v

    P RT v

    P RT v

    P RT visothermal

    I

    Combine the two steps:

    = = K molJ 354.0lnln 12

    1

    2PP R

    T T cs Psys

    (c)For this construction, we use the path diagrammed on the followingPT diagram:

  • 8/13/2019 felix termodinamica quimica ch03

    29/104

    29

    20

    40

    490 560

    1

    2

    T

    P

    Isochoric heating

    Isothermalcompression

    I

    For isochoric heating followed by an isothermal expansion, the entropy can be expressed asfollows:

    isothermalisochoricsys sss +=

    For the reversible, isothermal expansion, we obtain (refer to Parts (a) and (b) to see how this isderived)

    =

    I isothermal P

    P Rs 2ln

    However, we can relate the intermediate pressure to the initial pressure through the ideal gas law

    11

    2 T P

    T P I =

    or

    [bar]4.27= I P

    =

    =

    =

    12

    12

    1

    1

    222 lnlnlnln

    PP

    RT T

    RP

    T T

    P R

    PP

    Rs I

    isothermal

    For the isochoric process:

    dT cq vrev =

    Therefore,

  • 8/13/2019 felix termodinamica quimica ch03

    30/104

  • 8/13/2019 felix termodinamica quimica ch03

    31/104

    31

    3.13

    (a)Since the process is reversible and adiabatic, the entropy change for the process is zero.

    0= syss Furthermore, since the process is adiabatic, the changes in entropy of the surroundings and theuniverse are also zero.

    (b)

    Since this process is isentropic (s=0), we can apply an expression for the entropy change of anideal gas.

    s =0 = cP

    T T 1

    T 2

    dT R ln P2P1

    Insert the expression for cP from Appendix A

    +==

    1

    2263

    ln10227.010506.0639.302

    1PP

    RdT T

    T T Rs

    T

    T

    which upon substituting

    [ ][ ]

    [ ] bar 06.12 bar 1

    K 250

    2

    1

    1

    ===

    P

    P

    T

    yields

    [ ]K 4822 =T

    (c)An energy balance gives

    ( ) ===2

    1

    2

    1

    1T

    T

    T

    T

    Pv dT cdT cuw

  • 8/13/2019 felix termodinamica quimica ch03

    32/104

  • 8/13/2019 felix termodinamica quimica ch03

    33/104

    33

    3.14The problem statement states that the vessel is insulated, so we can assume that heat transfer tothe surroundings is negligible. Therefore, the expression for the entropy change of the universis

    sysuniv ss =

    An energy balance will help us solve for the entropy change. Neglecting potential and kineticenergy effects, the energy balance is

    W QU +=

    Since the vessel is insulated, the heat term is zero. Furthermore, no work is done, so the energy balance is

    ( ) 0 12 == uumU 21 uu =

    The values for the initial pressure and temperature constrain the value of specific energy at state1. From the steam tables,

    =kgkJ 2.26191u (400 C, 200 bar)

    =kgkJ 2.26192u

    The problem statement provides the pressure of state 2 (100 bar). Since we know the pressureand internal energy at state 2, the entropy is constrained. Information given in the problemstatement also constrains state 1. From the steam tables,

    =K kg

    kJ 5539.51s (400 C, 200 bar)

    =K kg

    kJ 7754.52s

    =kgkJ 2.2619 bar,100 2u

    Therefore,

    ===K kg

    kJ 2215.0 12 ssss sysuniv

    ==K kJ 2215.0sysuniv S S

  • 8/13/2019 felix termodinamica quimica ch03

    34/104

  • 8/13/2019 felix termodinamica quimica ch03

    35/104

    35

    3.16

    (a) A schematic of the process is shown below:

    1 bar, 298 K

    MixingProcess

    N2 N2

    N2

    N2

    O2

    O2

    N2

    N2

    N2

    N2

    N2

    N2 N2

    N2

    O2

    N2 N2

    N2

    N2

    O2

    1 bar, 298 K P2 , T 2

    The tank is insulated. The change in entropy of the universe can be rewritten as

    22 ,, Osys N sysuniv S S S +=

    since the tank is well-insulated. After the partition ruptures, the pressure and temperature willremain constant at 1 bar and 298 K, respectively. This can be shown by employing mass andenergy balances. Therefore, the partial pressures in the system are

    bar 79.02,2 = N p

    bar 21.02,2 =O p

    Use Equation 3.62 to determine the entropies:

    =

    = bar 1

    bar 79.0lnK mol

    J314.8ln2

    2 12

    K 298

    K 298,

    N

    P N sys P

    P RdT

    T c

    s

    =K mol

    J96.12, N syss

    =

    = bar 1

    bar 21.0lnK mol

    J314.8ln2

    2 12

    K 298

    K 298,

    O

    POsys P

    P RdT

    T c

    s

    = K molJ98.122,Osyss

    Therefore,

    ( ) ( )

    +

    =

    K molJ98.12mol21.0

    K molJ96.1mol79.0univS

  • 8/13/2019 felix termodinamica quimica ch03

    36/104

    36

    =K J27.4univS

    (b) A schematic of the process is shown below:

    2 bar, 298 K

    MixingProcess

    N2

    N2

    N2

    N2 N2

    N2 N2

    N2

    O2 N2

    N2

    N2

    N2 N2

    N2 N2

    N2

    O2

    N2

    N2

    N2

    N2

    N2

    N2 N2

    N2

    O2

    N2

    N2

    N2

    N2 N2

    N2

    N2

    N2

    O2

    1 bar, 298 K P2 , T 2

    Before calculating the change in entropy, we need to find how the temperature and pressure

    change during the process. The energy balance simplifies to0=U

    which can be rewritten as

    ( ) ( ) 0K 298K 2982222 2 =+ vOv N v N O cncnT cnn

    Assuming the heat capacities are equal, we can show that

    K 2982 =T

    We can find the pressure after the rupture by recognizing that the tank is rigid. Therefore,

    tot N O V V V =+ 22

    By employing the ideal gas law, we get the following equation (it has been simplified):

    2,1,122

    2

    2

    2

    2

    P

    nn

    P

    n

    P

    n N O

    O

    O

    N

    N +=+

    Substitute values and solve to obtain

    bar 65.12 =P

    Now, use Equation 3.62 to calculate the entropies as was done in Part (a):

  • 8/13/2019 felix termodinamica quimica ch03

    37/104

    37

    =

    = bar 2

    bar 30.1lnK mol

    J314.8ln2

    2 12

    K 298

    K 298,

    N

    P N sys P

    P RdT

    T c

    s

    =K mol

    J 58.32, N syss

    =

    = bar 1

    bar 347.0lnK mol

    J314.8ln2

    21

    2K 298

    K 298,

    O

    POsys P

    P RdT

    T c

    s

    =K mol

    J8.82,Osyss

    Therefore,

    ( ) ( )

    +

    =K mol

    J8.8mol21.0K mol

    J58.3mol79.0univ

    S

    =K J68.4univS

  • 8/13/2019 felix termodinamica quimica ch03

    38/104

  • 8/13/2019 felix termodinamica quimica ch03

    39/104

    39

    3.18For this process to work, conservation of mass and the first and second laws of thermodynamicmust hold. The subscript 1 refers to the inlet stream, 2 refers to the cold outlet, and 3refers to the hot outlet. To test the conservation of mass, perform a mass balance

    321 mmm &&& +=

    +=s

    kg 5.1s

    kg 5.0s

    kg 2

    Clearly, the conservation of mass holds. Now test the first law of thermodynamics by writing aenergy balance. Since there is no heat transfer or work, the energy balance becomes

    0 113322 =+ hmhmhm &&& (PE and KE effects neglected)

    Using the conservation of mass we can rewrite the mass flow rate of stream 1 in terms of stream2 and 3:

    ) ) 0 133122 =+ hhmhhm && If we assume that the heat capacity of the ideal gas is constant, the equation can be written asfollows:

    ( ) ( ) 0 133122 =+ T T cmT T cm PP &&

    Therefore,

    0K)20(s

    kg 5.1K)60(s

    kg 5.0 =+ PP cc

    This proves that the first law holds for this system. For the second law to be valid, the rate ofchange in entropy of the universe must greater than or equal to 0, i.e.,

    dS dt

    univ

    0

    Assuming the process is adiabatic, we can write the rate of entropy change of the universe forthis steady-state process using Equations 3.48-3.50:

    ( ) ( )133122113322 ssmssmsmsmsmdt dS

    univ

    +=+=

    &&&&&

  • 8/13/2019 felix termodinamica quimica ch03

    40/104

    40

    where a mass balance was used. For constant heat capacity, we can calculate the entropydifferences using Equation 3.63:

    =

    =

    1

    2

    1

    2

    1

    2

    1

    212 lnln2

    5lnlnPP

    T T

    RPP

    RT T

    css P

    =

    =

    1

    3

    1

    3

    1

    3

    1

    313 lnln2

    5lnlnPP

    T T

    RPP

    RT T

    css P

    Applying these relations, we get:

    ( )Puniv

    c R MW dt

    dS 057.077.21 =

    Therefore, the second law holds if RcP

    6.48 ; this value is far in excess of heat capacity forgases, so this process is possible.

  • 8/13/2019 felix termodinamica quimica ch03

    41/104

    41

    3.19

    (a)

    T 1 = 640oC

    V 1= 20 m/s Nozzle V 2=?

    P1 = 4 MPa P 2 = 100 kPa

    (b)Since the process is reversible and adiabatic,

    =K J 0sysS

    (c)

    The steam tables can be used to determine the final temperature. From the interpolation of steatable data

    ( ) == K kgkJ 4692.7MPa4C,640 12 ss

    Now two thermodynamic properties are known for state 2. From the steam tables:

    C4.1212 =T

    ==K kg

    kJ 4692.7MPa,1.0 22 sP

    (d)An energy balance is required to calculate the exit velocity. The energy balance for the nozzle

    ( ) ( )1122 0 K K ehmehm ++= &&

    Realizing that 21 mm && = allows the energy balance to be written as

    ( ) 212, hehe K K +=

    which is equivalent to

    ( )[ ]2122 5.02 hV hV += rr

    Substituting the following values

  • 8/13/2019 felix termodinamica quimica ch03

    42/104

    42

    =sm 201V

    r

    =kgJ 27191002h (0.1 MPa, 121.4 C)

    = kgJ 37670001h (4 MPa, 640 C)

    yields

    =sm 14482V

    r

    Note: the velocity obtained is supersonic; however, the solution does not account for this type oflow.

  • 8/13/2019 felix termodinamica quimica ch03

    43/104

    43

    3.20

    A schematic of the process follows:

    Propane inv1 = 600 cm3/molT 1= 350oC

    Turbine

    P2 = 1 atm

    ws

    Propaneout

    Since this process is isentropic (s=0), we can apply an expression for the entropy change of anideal gas. We must be careful, however, to select an expression which does not assume a

    constant heat capacity.

    s =0 = cPT T 1

    T 2 dT R ln P2

    P1

    Insert the expression for cP from Appendix A

    +==

    1

    2263

    ln10824.810785.28213.102

    1PP

    RdT T

    T T Rs

    T

    T

    We can also relate P1 to T1 and v1 (which are known) through the ideal gas law:

    P1 = RT 1

    v1

    This leaves us with 1 equation and 1 unknown (T2). Integrating:

    ( ) + ==

    1

    1221226123

    1

    2 ln10412.4)(10785.28ln213.10 RT vPT T T T

    T T s

    UsingT1 = 623 K,v1 = 600 cm3/mol,P2 = 1 atm

  • 8/13/2019 felix termodinamica quimica ch03

    44/104

    44

    R=82.06 cm3 atm/mol K,

    we get:T2 = 454 K

    To solve forn

    W S &

    & we need a first law balance, with negligible ke and pe, the 1st law for a steady

    state process becomes:

    ( ) S W Qhhn &&& ++= 210

    If heat transfer is negligible (isentropic),

    ( ) +===2

    1

    2

    1

    26312 10824.810785.28213.1

    T

    T

    T

    T p

    S dT T T RdT chhn

    W &

    &

    integrating:

    ( ) ( ) ( )[ ]313262122312 10941.210393.14213.1 T T T T T T RnW S +=

    &

    &

    Substituting

    K 623TK 454

    K molJ 314.8

    1

    2==

    =

    T

    R

    results in

    =mol

    J 19860n

    W S &

    &

  • 8/13/2019 felix termodinamica quimica ch03

    45/104

  • 8/13/2019 felix termodinamica quimica ch03

    46/104

    46

    Therefore,

    +

    ==22 1

    2

    1

    2 lnln N O

    surr surr surr surr PP

    PP

    RT sT q

    ( )

    +

    =

    22 bar 79.0

    bar 1ln bar 21.0

    bar 1lnK 15.293K mol

    J 314.8 N O

    surr q

    =mol

    J 4378.2surr q

    and

    =mol

    J 4378.2q

    Energy Balance:

    0=++out

    S out out in

    inin W Qhnhn &&&&

    Because the temperature of the oxygen and nitrogen doesnt change in the process, the energy balance per mole of feed becomes

    qwS =

    =mol

    J 4378.2S w

  • 8/13/2019 felix termodinamica quimica ch03

    47/104

    47

    3.22

    (a)Take the entire container to be the system and assume no heat or work crosses the system boundary.

    Energy Balance:

    0=U

    After the oscillation cease, the temperature and pressure on both sides of the piston will be thesame assuming the metallic piston is very thin and the thermal conductivity coefficient is large. Now, lets rewrite the energy balance.

    0Bgas,1Agas,1 =+= ununU B A

    or

    0BgasAgas,1

    ,1 =+ uun

    n

    B

    A

    41.22 1,1,

    1,1,

    1,1,1,

    1,1,1,

    ,1

    ,1 === A B

    B A

    A B B

    B A A

    B

    A

    T P

    T P

    T V P

    T V P

    n

    n

    since

    1,1,2 B A V V = so

    ( ) ( ) 0101.55/2K 15.7732341.2

    2T

    K 15.373

    3-2 =++ dT T RT R

    K 5852 =T

    Mass Balance:

    2,2,1,1, B A B A nnnn +=+

    Using the ideal gas law, we get

  • 8/13/2019 felix termodinamica quimica ch03

    48/104

    48

    2

    2

    2,

    2,2,

    2,

    2,2,

    1,

    1,1,

    1,

    1,1,

    RT V P

    RT

    V P

    RT

    V P

    RT

    V P

    RT

    V P tot B

    B B

    A

    A A

    B

    B B

    A

    A A =+=+

    since the pressure and temperature of state 2 are equal on both sides. But

    1,1,1, 3 A B Atot V V V V =+=

    Using the volume relationships and simplifying, we get

    [ ] [ ][ ]

    [ ][ ]

    +=

    +=

    K 15.373 bar 12

    K 15.773 bar 10

    3K 8552

    3 1,1,

    1,

    1,22

    B

    B

    A

    A

    T

    P

    T

    PT P

    [ ] bar 56.32 =P

    (b)The container is well-insulated, so

    sysuniv ss =

    The entropy change of the system can be split into two subsystems:

    Bgas,1,1

    ,1Agas

    ,1,1

    ,1 snn

    ns

    nn

    ns

    B A

    B

    B A

    Asys +

    ++

    =

    Using Equation 3.63 and realizing that Rcc vP += ,

    =

    =

    K molJ 96.1lnln5.2

    41.341.2

    1,

    2

    1,

    2Agas

    A A PP

    RT T

    Rs

    =

    +=

    K molJ 51.1ln105.15.3

    41.31

    1,

    23

    Bgas

    2

    1, B

    T

    T PP

    RdT T

    T Rs

    B

    Therefore,

    =+==K mol

    J 47.3K mol

    J 51.1K mol

    J 96.1sysuniv ss

    The process is possible.

  • 8/13/2019 felix termodinamica quimica ch03

    49/104

    49

    3.23

    Equation 3.3 can be modified to show

    surr sysuniv S S S +=

    Lets first calculate sysS . Before this problem is solved, a few words must be said about thenotation used. The system was initially broken up into two parts: the constant volume containeand the constant pressure piston-cylinder assembly. The subscript 1 refers to the constantvolume container; 2 refers the piston-cylinder assembly. i" denotes the initial state before thvalve is opened, and f denotes the final state.

    First, the mass of water present in each part of the system will be calculated. The mass will beconserved during the expansion process. Since the water in the rigid tank is saturated and is inequilibrium with the constant temperature surroundings (200 C), the waters entropy isconstrained. From the steam tables,

    [ ] kPa8.1553kg

    kJ 4322.6

    kgkJ 3308.2

    kgkJ 12736.0

    kgkJ 001156.0

    ,1

    ,1

    ,1

    ,1

    =

    =

    =

    =

    =

    sat

    v

    i

    li

    vi

    li

    P

    s

    s

    v

    v

    (Sat. water at 200 C)

    Knowledge of the quality of the water and the overall volume of the rigid container can be usedto calculate the mass present in the container.

    ( ( v il i vmvmV ,11,111 95.005.0 +=

    Using the values from the steam table and 31 m5.0=V provides

    [ ]kg13.41 =m

    Using the water quality specification,

  • 8/13/2019 felix termodinamica quimica ch03

    50/104

    50

    [ ][ ]kg207.005.0kg92.395.0

    11

    11

    ==

    ==

    mm

    mml

    v

    For the piston-cylinder assembly, bothP andT are known. From the steam tables

    =

    =

    K kgkJ 9665.6

    kgm 35202.0

    ,2

    3,2

    i

    i

    s

    v

    (600 kPa, 200 C)

    Now enough information is available to calculate the mass of water in the piston assembly.

    [ ]kg284.02

    22 ==

    v

    V m

    Now the final state of the system must be determined. It helps to consider what physicallyhappens when the valve is opened. The initial pressure of the rigid tank is 1553.8 kPa. Whenthe valve is opened, the water will rush out of the rigid tank and into the cylinder untilequilibrium is reached. Since the pressure of the surroundings is constant at 600 kPa and thesurroundings represent a large temperature bath at 200 C, the final temperature and pressure othe entire system will match the surroundings. In other words,

    ==kgkJ 9665.6 ,2 i f ss (600 kPa, 200 C)

    Thus, the change in entropy is given by

    li

    li

    vi

    vii f

    li

    visys smsmsmsmmmS ,1,1,1,1,22,1,12 )( ++=

    Substituting the appropriate values reveals

    =K kJ 05.3sysS

    Now we calculate the change in entropy of the surroundings. Since the temperature of thesurroundings is constant,

    surr surr

    surr surr T

    QT Q

    S ==

    After neglecting potential and kinetic energy effects, the energy balance becomes

  • 8/13/2019 felix termodinamica quimica ch03

    51/104

    51

    W QU +=

    The change in internal energy and work will be calculated in order to solve forQ. The followingequation shows how the change in internal energy can be calculated.

    li

    li

    vi

    vii f

    li

    vi umumumummmU ,1,1,1,1,22,1,12 )( ++=

    From the steam tables

    =

    =

    kgkJ 3.2595

    kgkJ 64.850

    ,1

    ,1

    vi

    li

    u

    u

    (sat. H2O at 200 C)

    ==kgkJ 9.2638 ,2 i f uu (600 kPa, 200 C)

    Using these values and the values of mass calculated above,

    [ ]kJ0.541=U

    Calculating the work is relatively easy since the gas is expanding against a constant pressure of600 kPa (weight of the piston was assumed negligible). From Equation 2.7,

    )( i f E V

    V E V V PdV PW

    f

    i

    ==

    where

    [ ][ ]

    [ ] [ ] [ ]3333

    ,2,1,12

    m6.0m5.0m0.1

    m55.1)(

    Pa600000

    =+=

    =++==

    i

    il

    iv

    i f

    E

    V

    vmmmV

    P

    Note: iv ,2 was used to calculate f V because the temperature and pressure are the samefor the final state of the entire system and the initial state of the piston-cylinder assembly

    The value ofW can now be evaluated.

    [ ]kJ570=W

  • 8/13/2019 felix termodinamica quimica ch03

    52/104

    52

    The energy balance can be used to obtainQ.

    [ ] [ ]( ) [ ]kJ1111kJ570kJ0.541 === W U Q

    Therefore,[ ][ ]

    ==K kJ 35.2

    K 15.473kJ1111

    surr S

    and

    ==+=K kJ 70.0

    K kJ 35.2

    K kJ 05.3surr sysuniv S S S

  • 8/13/2019 felix termodinamica quimica ch03

    53/104

    53

    3.24Let the subscript 1 represent the state of the system before the partition is removed. It has twcomponents: componenta and componentb. Subscript 2 represents the system after the partition is removed.

    Mass balance:2,1,1 nnn ba =+

    Energy balance for the adiabatic process:

    W U sys = ) ) )babbvbaava V V V PT T cnT T cn ,1,12,12,,1,12,,1 +=+

    The external pressure,P, for the above energy balance is equal toP 1,a . To find the final

    temperature, first find the volumes using the ideal gas law.

    [ ]( ) ( )

    [ ]( ) [ ]( )[ ]

    [ ]32

    2-,1m05.0

    m098.0sm8.9kg1000

    K 300K mol

    J 314.8mol2=

    =aV

    [ ]( )

    [ ]( ) [ ]( )[ ][ ]324

    22-

    22 m103256.3

    m098.0sm8.9kg1000

    K molJ 314.8mol4

    T

    T

    V =

    =

    Substitute the volumes into the energy balance and solve for2T :

    ( ) ( ) ( ) ( ) ( )[ ]324522 m15.0103256.3Pa101K 30023mol2K 300

    23mol2 =

    +

    T T RT R

    K 4.3602 =T

    To calculate the change in entropy, we can use the following relationship

    bPb

    aPasys P

    P R

    T T

    cnPP

    RT T

    cnS

    +

    =

    12

    12

    12

    12 lnlnlnln

    The only unknown in the above equation isP 1,b , so we can calculate it with the ideal gas law:

  • 8/13/2019 felix termodinamica quimica ch03

    54/104

  • 8/13/2019 felix termodinamica quimica ch03

    55/104

    55

    3.25First perform an energy balance on the process:

    W U sys =

    The change in internal energy can be written:

    unununU i A f B f B f A f A ,,,,, +=

    A mass balance gives:

    f B f Ai A nnn ,,, +=

    These two expressions can be substituted to give:

    ) ) W T T cnT T cnU i f Bv f Bi f Av f A =+= ,,,,

    Using the ideal gas law and Rcv 25= , we obtain

    W V PV PV P i Ai A f A f A f B f B =+ ,,,,,, 25

    25

    25

    Now, find an expression for the work

    =

    f

    i

    V

    V E

    dV PW

    The pressure is not constant; its value is given by

    Amg

    A

    V V k P

    Amg

    Akx

    PP i B Batmatm E ++=++= 2

    ,

    [ ]Pa10284.81006.1 55 B E V P +=

    Integrating10142.41006.1 2

    ,

    5

    ,

    5 f B f B

    V V W =

    Substituting this expression into the energy balance gives:

    2,

    5,

    5,,,,,, 10142.41006.12

    525

    25

    f B f Bi Ai A f A f A f B f B V V V PV PV P =+

  • 8/13/2019 felix termodinamica quimica ch03

    56/104

  • 8/13/2019 felix termodinamica quimica ch03

    57/104

  • 8/13/2019 felix termodinamica quimica ch03

    58/104

    58

    =kgkJ 9.28272h

    Energy balance:

    ( ) sW Qhhm &&& += 12

    CalculateW s:

    ( ) ( ) ( ) [ ]kW86.69kgkJ1.3422

    kgkJ9.2827kg/s25.1 12

    == QhhmW s &&&

    [ ]kW673=sW &

    (c)The isentropic efficiency is defined as follows:

    ( )( )reversibleW

    W

    S

    actualS &

    &=

    For this situation,

    ( ) [ ]( ) [ ]kW5.447kW673665.0 ==actualS W &

    (d)The real temperature should be higher since not as much energy is converted into work.

    (e)Use the energy balance:

    ) sW Qhhm &&& += 12 =++=++=

    kgkJ 2.3008

    kgkJ1.3422

    kg/s25.1kW-447.5kW86.69 12 hm

    W Qh s

    &

    &&

    At 1 MPa, water has this value of enthalpy when

    ( ) C2.2802 =actualT

  • 8/13/2019 felix termodinamica quimica ch03

    59/104

    59

    3.27Since the compressor is adiabatic, the energy balance after neglecting potential and kineticenergy becomes

    ( ) S W hhn && = 12

    Using the ideal gas law and Appendix A.2, the above equation becomes

    ( ) ++++= 2

    1

    322111

    T

    T S dT ET DT CT BT AT

    V PW

    &&

    Substituting the following values

    [ ] [ ]Pa101 bar 1 51 ==P

    =s

    m 1 31V &

    [ ]K 15.2931 =T [ ]K 15.4732 =T

    ;355.3= A ;10575.0 3= B ;0=C ;10016.0 5= D 0= E (Table A.2.2)

    gives

    [ ]kW8.218=S W &

    This is the work of our real turbine (with 80% isentropic efficiency). We can use the isentropicefficiency to calculate the work of an equivalent 100% efficient process.

    ( )( )compressor S

    reversibleS compressor W

    W &

    &=

    ( ) ( ) ( ) [ ]( )kW8.2188.0== compressor compressor S reversibleS W W && ( ) [ ]kW75.41=reversibleS W &

    Since the process is adiabatic, we can use the following equation again to calculate what the fintemperature would be in a reversible process.

    ( )2

    1

    322

    111 ++++=

    T

    T S dT ET DT CT BT AT

    V PW

    &&

  • 8/13/2019 felix termodinamica quimica ch03

    60/104

    60

    [ ] ( ) ++++= 2

    1

    W175400 322111

    T

    T

    dT ET DT CT BT AT V P &

    Substituting the values from above and solving for2T , we obtain

    K8.164,2 =revT

    To obtain the pressure the final pressure for the real compressor, we can calculate the final pressure for the reversible process because the final pressure is the same in both cases. For theisentropic expansion

    ==

    12ln

    T0

    2

    1PP

    RdT c

    sT

    T

    p

    ++++==

    12

    322ln0

    2

    1PP

    RdT T

    ET DT CT BT A Rs

    T

    T

    ( )[ ]

    +==

    =

    =

    bar 1ln160010575.0355.30 2

    K 438

    K 15.293

    232

    1

    PdT

    T T T

    RsT

    T

    Solve for P2:

    [ ] bar 16.42 =P

  • 8/13/2019 felix termodinamica quimica ch03

    61/104

    61

    3.28Isentropic efficiency for a turbine is defined as

    ( )( )revs

    actuals

    w

    w=

    If the rate of heat transfer is assumed negligible, the energy balance for this process is

    sW hhm && += 21 0

    For a reversible, adiabatic process,

    0= syss ( )MPa10C,500 1,2 ss rev =

    From the steam tables

    ==K kg

    kJ 5965.6 1,2 ss rev

    Since sat vsat l sss ,22,

    2 , a mixture of liquid and vapor exists. The quality of the water can becalculated as follows

    ( ) sat vsat lrev s xs xs ,2,2,2 1 +=

    ( )

    +

    =

    K kgkJ .35937

    K kgkJ .302511

    K kgkJ 5965.6 x x

    874.0= x

    Therefore,

    ( ) sat vsat lrev h xh xh ,2,2,2 1 +=

    ( )

    +

    =kgkJ 675.52874.0

    kgkJ 17.444874.01 ,2 revh

    =kgkJ 0.2391 ,2 revh

    Also, from the steam tables,

    =kgkJ 6.33731h (500 C, 10 MPa)

  • 8/13/2019 felix termodinamica quimica ch03

    62/104

  • 8/13/2019 felix termodinamica quimica ch03

    63/104

  • 8/13/2019 felix termodinamica quimica ch03

    64/104

    64

    Substitute these expressions into the entropy balance and solve forP 1:

    bar 85.11 =P

  • 8/13/2019 felix termodinamica quimica ch03

    65/104

  • 8/13/2019 felix termodinamica quimica ch03

    66/104

    66

    =kgkJ 12.31561u (540 C, 60 bar)

    Hence,

    [ ]( ) [ ]kJ1373kgkJ 12.3156

    kgkJ 15.2881kg5 =

    =W

    (d)The specific volume can be found in the steam tables

    =kgm 14173.0

    32v

    Therefore,

    [ ]( ) [ ]332 m709.0kgm 14173.0kg5 =

    =V

  • 8/13/2019 felix termodinamica quimica ch03

    67/104

    67

    3.31A schematic of the process is shown below:

    well-insulated

    very tinyleak hole

    P1 = 60 bar

    m = 5 kg

    Pure H2O

    T 1 = 540oCP2 = 20 bar

    Pure H2O

    T ' 2 = ?oC

    T surr = 25oCPsurr = 1 bar

    In order to leave the system, the gas must do flow work on the surroundings. The initial state ithe same as for Problem 3.30 and the final pressures are the same. Since the water only expandagainst 1 bar, the work is lower than that for the differential process described in Problem 3.30.Thus, this adiabatic process looses less energy, leading to a higher final temperature.

    Another way to view this argument is to look at this process as a closed system. This depictionis the expansion analog of the compression process depicted for Example 2.5 in Figure E2.5B(page 57). We can represent this process in terms of two latches, one that keeps the process in initial state at 60 bar and one that stops the expansion after the pressure has reduced to 20 bar.The process is initiated by removal of the first latch and ends when the piston comes to restagainst the second latch. Such a process is depicted as Problem 3.31 below. Thecorresponding reversible process of Problem 3.30 is shown next to it for comparison. Clearly th process on the left does less work, resulting in a greater final temperature.

    Problem 3.31 Problem 3.30

    P1 = 60 bar

    m = 5 kg

    Pure H2O

    T 1 = 540oC

    P2 = 20 bar

    m = 5 kg

    Pure H2O

    T surr = 25oCPsurr = 1 bar

    irreversible

    latches

    T ' 2 = ?oC

    P1 = 60 bar

    m = 5 kg

    Pure H2O

    T 1 = 540oC

    P2 = 20 bar

    m = 5 kg

    Pure H2O

    T 2 = ?oC

    T surr = 25oCPsurr = 1 bar

    well-insulated

    reversible

  • 8/13/2019 felix termodinamica quimica ch03

    68/104

    68

    3.32

    (a)Consider the tank as the system.

    Mass balance

    inout in mmmdt dm

    &&& ==

    Separating variables and integrating:

    =t

    in

    m

    m

    dt mdm0

    2

    1

    &

    or=t

    in dt mmm0

    12 &

    Energy balanceSince the potential and kinetic energy effects can be neglected, the open system, unsteady stateenergy balance is

    ++=

    out s

    inininout out

    sysW Qhmhm

    dt dU &&&&

    The process is adiabatic and no shaft work is done. Furthermore, there is only one inlet streamand not outlet stream. Therefore, the energy balance simplifies to

    ininsys

    hmdt

    dU &=

    The following math is performed

    in

    t

    inin

    t

    inin

    U

    U

    hmumU

    dt mhdt hmdU

    2222

    000

    2

    1

    ==== =

    &&

    where the results of the mass balance were used. Thus,

    inhu 2 =

  • 8/13/2019 felix termodinamica quimica ch03

    69/104

    69

    From the steam tables,

    =kgkJ 5.3456inh (3 MPa, 773 K)

    Now the water in the tank is constrained. From the steam tables:

    =

    =

    kgm14749.0

    K kgkJ 743.7

    32

    2

    v

    s

    =kgkJ 5.3456 MPa,3 2u

    Compute the change in entropy. An entropy balance gives:

    ininsysuniv

    smdt dS

    dt dS

    &

    =

    Integrating withsin constant

    ( )int

    ininuniv ssmdt mssmS 220

    22 == &

    From the steam tables:

    =K kg

    kJ 2337.7ins (3 MPa, 773 K)

    Therefore,

    =

    =K kJ 173.0

    K kgkJ 2337.7

    K kgkJ 743.7

    kgm

    14749.0

    m05.03

    3univS

    (b)If it tank sits in storage for a long time and equilibrates to a final temperature of 20 C, some orall of the vapor will condense and exchange heat with the surroundings. Let the subscript 3designate the final state of the water when it has reached a temperature of 20 C. The change inentropy between state 2 and state 3 is given by the following equation

  • 8/13/2019 felix termodinamica quimica ch03

    70/104

    70

    ( )T

    QssmS surr univ += 232

    Let the subscript 3 designate the final state of the water when it has reached a temperature of

    20 C. First, find quality of the water. From the steam tables:

    =

    =

    kgm79.57

    kgm001002.0

    3,3

    3,3

    sat v

    sat l

    v

    v

    (sat. water at 20 C)

    Calculate the quality as follows:

    ( ) sat vsat l v xv xvv ,3,323 1 +==

    ( )

    +

    =

    kgm79.57

    kgm 001002.01

    kgm14749.0

    333

    x x

    0025.0= x

    Now calculate the entropy of state 3

    ( ) sat vsat l s xs xs ,3,33 1 +=

    Substitute values from the steam tables:

    ( )

    +

    =K kg

    kJ6671.80025.0K kg

    kJ 2966.00025.013s

    =K kg

    kJ .317503s

    Now calculate the amount of heat transferred to the surroundings:

    ( )232 uumQQ surr ==

    Calculate the internal energy of state 3:

    ( ) sat vsat

    l u xu xu ,3,33 1 +=

    Substituting values from the steam tables:

  • 8/13/2019 felix termodinamica quimica ch03

    71/104

    71

    =kgkJ 74.893u

    Therefore,

    [ ]kJ1141kgkJ 5.3456

    kgkJ 89.74

    kgm 14749.0

    m05.03

    3=

    =surr Q

    Now calculate the change in entropy of the universe

    [ ]K 293kJ1411

    K kgkJ 743.7

    K kgkJ .31750

    kgm 14749.0

    m05.03

    3

    +

    = univS

    =K kJ 1.38univS

    The entropy change for both processes can be fount by adding together the entropy change fromPart (a) and Part (b):

    ( ) =K kJ1.55(b)and(a)univS

  • 8/13/2019 felix termodinamica quimica ch03

    72/104

    72

    3.33A schematic is given below

    valve maintainspressure in system

    constant

    T 1 = 200 oCx 1 = 0.4V = 0.01 m 3

    v

    l

    Mass balance

    out out in mmmdt dm

    &&& ==

    Separating variables and integrating:

    =t

    out

    m

    m

    dt mdm0

    2

    1

    &

    or=t

    out dt mmm0

    12 &

    Entropy Balance:

    surr out out

    syssurr

    surr out out

    sysuniv T

    Qsm

    dt dS

    T Q

    smdt dS

    S &

    &&

    && +

    =++

    =

    Integrating withsout constant

    ( )surr

    out univ T Q

    smmsmsmS = 121122 (1)

    From steam tables:

  • 8/13/2019 felix termodinamica quimica ch03

    73/104

    73

    =+=+=K kg

    kJ 9715.34323.64.03309.26.0)1(1 g f xuu xs

    =K kg

    kJ 4323.62s is the same as 2K kg

    kJ 4323.6 ssout ==

    Thus Equation 1 simplifies to

    surr univ T

    QssmS = )( 121 (2)

    Energy Balance to findQ :

    Qhmdt dU

    out out sys

    && +=

    Integrating

    [ ] +=+=t t t

    out out out out

    um

    um

    dt Qdt mhdt QhmdU 0 00

    22

    11

    &&&&

    Substituting in the mass balance and solving forQ

    ( ) out hmmumumQ

    121122 = To find the mass in each state:

    =+=+=kgm 0.0511274.04.0001.6.0)1(

    31 g f xvv xv

    =kgm 1274.0

    32v

    [ ]kg196.0kgm 0.051

    m01.03

    3

    11

    1 === vV

    m and [ ]kg0785.0kgm 0.1274

    m01.03

    3

    22

    2 === vV

    m

    From the seam tables:

  • 8/13/2019 felix termodinamica quimica ch03

    74/104

    74

    =+=+=kgkJ 54915.25974.064.8506.0)1(1 g f xuu xu

    =kg

    kJ 3.25952u and =kg

    kJ 2.2793out h

    Plugging in:

    ( ) [ ]kJ228 121122 == out hmmumumQ

    Back into Equation 2

    ( ) 0

    s

    K 473

    skJ 228

    K kgkJ 9715.34323.6

    skg 196.0)( 121 =

    ==

    surr univ T

    QssmS

    Note: Vaporization is reversible ifT surr = T sys.

  • 8/13/2019 felix termodinamica quimica ch03

    75/104

    75

    3.34The process can be represented as:

    State 1 State 2

    V 1 = 1 m3 V 2 = 10 m3P

    1 = 10 bar

    T 1 = 1000 K Work out

    Process

    Q = 0

    P2

    = ?T 2 = ?

    Solving for the number of moles:

    n = P1V 1 RT 1

    =120.3 mol

    The maximum work is given by a reversible process. Since it is also adiabatic, the entropychange of the system is zero:

    s =0=cP ln T 2T 1

    R ln P2P1

    SincecP =52

    R

    12

    2

    1

    22/5

    1

    2

    PV nRT

    PP

    T T ==

    Solving forT 2 gives

    ( ) K 2153/2

    12

    2/51

    2 == PV T nR

    T

    An energy balance on this closed system gives:

    U = Q +W =W =n c vdT = n cP R( )dT T 1

    T 2

    T 1T 2

    Solving for work, we get

    W = n 32

    R T 2 T 1( )= 1.18106 J

  • 8/13/2019 felix termodinamica quimica ch03

    76/104

    76

    3.35 The maximum efficiency is obtained from a Carnot cycle. From Equation 3.32

    H

    C

    T T

    n =1

    where temperature is in Kelvin. Hence,

    614.0K 15.773K 15.2981 ==n

  • 8/13/2019 felix termodinamica quimica ch03

    77/104

    77

    3.36The labeling described in Figure 3.8 will be used for this solution. First, a summary of knownvariables is provided.

    State P (bar) T (C)

    1 30 5002 0.134

    From our knowledge of ideal Rankine cycles, the table can be expanded as follows

    State ThermodynamicState

    P (bar) T (C)

    1 Superheated Steam 30 5002 Sat. Liq. and Vapor 0.1

    3 Sat. Liquid 0.14 Subcooled Liquid 30

    The saturation condition constrains state 3. First, start with the turbine. Since the rate of heattransfer is negligible and the expansion occurs reversibly in an ideal Rankine cycle, thefollowing is known

    21 ss = and

    ( )12 hhmW s = && From the steam tables:

    =K kg

    kJ 2337.71s (500 C, 30 bar)

    =

    =

    K kgkJ 1501.8

    K kgkJ 6492.0

    2

    2

    v

    l

    s

    s

    (sat. H2O at 0.1 bar)

    = kgkJ 5.34561h (500 C, 30 bar)

    =

    =

    kgkJ 6.2584

    kgkJ 81.191

    2

    2

    v

    l

    h

    h

    (sat. H2O at 0.1 bar)

  • 8/13/2019 felix termodinamica quimica ch03

    78/104

  • 8/13/2019 felix termodinamica quimica ch03

    79/104

    79

    Now we find the rate of heat transfer for the evaporator. For the entire Rankine cycle, Equation3.84 gives

    0=+++=+ C s H C net net W W QQW Q &&&&&&

    We have

    C C s H QW W Q &&&& =

    Using the values calculated above:

    [ ]MW2.326= H Q&

    Equation 3.84:

    C S net W W W &&&

    = [ ]MW08.116=net W &

    The efficiency can be calculated with Equation 3.82

    [ ][ ]

    356.0MW02.326MW08.116 ===

    H

    net

    Q

    W &

    &

  • 8/13/2019 felix termodinamica quimica ch03

    80/104

    80

    3.37To make the Rankine cycle more efficient, we need to increase the area that represents the network in Figure 3.8. This can be done in a variety of ways:

    1. Increase the degree of superheating of steam in the boiler. This process is sketched in the

    upper left handTs diagram below. This change reduces the moisture content of steamleaving the turbine. This effect is desirable since it will prolong the life of the turbine;however, if the steam is heated too high, materials limitations of the turbine may need to beconsidered.

    2. Lower the condenser pressure. Lowering the pressure in the condenser will lower thecorresponding saturation temperature. This change will enlarge the area on theTs diagram,as shown on the upper right below. Thus, we may want to consider lowering the pressure below atmospheric pressure. We can achieve this since the fluid operates in a closed loop.However, we are limited by the low temperature off the heat sink that is available.

    s

    1

    2

    3

    4

    wnet

    T

    s

    1

    2new3new

    4new

    wnet

    T

    lower the condenser pressure

    s

    1new

    2new3

    4

    wnet

    T

    increase the superheating of steam in the boiler

    s

    1

    23

    4wnet

    T

    increase the boiler pressureTurbine 1

    Turbine 2

    5

    6

    2 stage turbine with reheat

  • 8/13/2019 felix termodinamica quimica ch03

    81/104

    81

    3. Increase the boiler pressure. This will increase the boiler temperature which will increase tharea as shown on the bottom leftTs diagram.

    4. We can be more creative about how we use the energy available in the boiler. One way is todivide the turbine into two stages, Turbine 1 and Turbine 2, and reheat the water in the boile

    between the two stages. This process is illustrated below.

    turbine1

    coolingwater

    FuelAir

    Boiler

    compressor

    condenser

    1

    4

    56

    Tur-bine

    2

    2

    3

    Rankine cycle with reheat

    W s W s

    W c

    Q H

    QC

    The pressure in Turbine 1 will be higher than the pressure in turbine 2. This process isschematically shown on the bottom rightTs diagram. This process leads to less moisturecontent at the turbine exit (desirable) and limits the temperature of the superheat (desirable)

  • 8/13/2019 felix termodinamica quimica ch03

    82/104

    82

    3.38The labeling described in Figure 3.8 will be used for this solution. First, a summary of knownvariables is provided.

    State P (bar) T (C)

    1 100 5002 134

    From our knowledge of ideal Rankine cycles, the table can be expanded as follows

    State ThermodynamicState

    P (bar) T (C)

    1 Superheated Steam 100 500

    2 Sat. Liq. and Vapor 13 Sat. Liquid 14 Subcooled Liquid 100

    The saturation condition constrains state 3. First start with the turbine (state 2). Since the rate heat transfer is negligible and the expansion occurs reversibly in an ideal Rankine cycle, thefollowing is known

    21 ss =

    From the steam tables,

    =K kg

    kJ 5965.61s (500 C, 100 bar)

    =

    =

    K kgkJ 3593.7

    K kgkJ 3025.1

    2

    2

    v

    l

    s

    s

    (sat. H2O at 1 bar)

    The quality of the water can be calculated as follows

    vl s xs xs 221 )1( +=

    By substituting the steam table data, we find

    874.0= x

  • 8/13/2019 felix termodinamica quimica ch03

    83/104

    83

    States 1, 2, and 3 are constrained; therefore, the enthalpies can be determined using the steamtables. The enthalpy of state 4 can be calculated from Equation 3.80:

    ( )3434 PPvhh l +=

    where ==kgm 001043.0

    33vvl since specific volume of liquids are insensitive to pressure

    changes. From the steam tables:

    State 1 2 3 4

    h kgkJ 240.83

    kgkJ 391.02

    kgkJ 17.444

    kgkJ 8.724

    Note: vl h xh xh 222 )1( +=

    Equation 3.84:

    C H net QQW &&& =

    Therefore,

    23412341 hhhhmhhmhhmW net +=+= &&&&

    and

    ( ) ( )[ ][ ]

    +

    =+

    =

    kgkJ 0.2391

    kgkJ 27.84

    kgkJ 5.417

    kgkJ 240.83

    kW10100

    3

    2341 hhhh

    W m net

    &&

    =s

    kg 3.116m&

    Now consider the non-ideal turbine and compressor. Use the definition of isentropicefficiencies:

    ( )( )reversibles

    actualsturbine W

    W &

    &=

    ( ) ( ) ( ) reversibleturbinereversibleS turbineactualS hhmW W 12 == &&&

  • 8/13/2019 felix termodinamica quimica ch03

    84/104

    84

    ( )( )actualC

    reversibleC compressor W

    W &

    &=

    ( ) ( ) ( )compressor

    reversible

    compressor

    reversibleC actualC

    hhmW W

    34 ==

    &&&

    Equation 3.84 is used to find the mass flow rate

    C snet W W W &&& =

    ( ) +=compressor

    reversiblereversibleturbinenet

    hhhhmW

    3412

    &&

    Substituting the enthalpy values from the table shown above and the efficiency values gives

    =s

    kg 147m&

    A higher flow rate is needed as compared to the reversible process.

  • 8/13/2019 felix termodinamica quimica ch03

    85/104

    85

    3.39The labeling shown in Figure 3.8 will be used for this solution. First, a summary of knownvariables is provided.

    State Thermodynamic

    State

    P (MPa)

    1 Sat. Vapor 1.72 0.734

    From our knowledge of ideal Rankine cycles, the table can be expanded as follows

    State ThermodynamicState

    P (MPa)

    1 Sat. Vapor 1.7

    2 Sat. Liq. And Vapor 0.73 Sat. Liquid 0.74 Subcooled Liquid 1.7

    The enthalpy of states 1 and 3 can be found using the NIST website. For states 2 and 4, use thefollowing expressions to find the enthalpies

    vl h xh xh 222 )1( += ( ) ( )34333434 PPvhPPvhh l +=+=

    To find x, use the fact that the turbine is isentropic.

    21 ss = vl s xs xs 221 )1( +=

    From the NIST website:

    =K mol

    J 38.1461s (sat. vapor at 1.7 MPa)

    =

    =

    K molJ 07.150

    K molJ

    45.90

    2

    2

    v

    l

    s

    s (sat. mixture at 0.7 MPa)

    By substituting the above values into the entropy relationship, we find

  • 8/13/2019 felix termodinamica quimica ch03

    86/104

  • 8/13/2019 felix termodinamica quimica ch03

    87/104

    87

    3.40

    (a)Using the information in the problem statement and our knowledge of ideal vapor compressioncycles, the following table can be created

    State ThermodynamicState

    P (MPa)

    1 Sat. Mixture 0.122 Sat. Vapor 0.123 Superheated Vapor 0.74 Sat. Liquid 0.7

    Note: Refer to Figure 3.9 to review labeling convention.

    Equation 3.85 states

    ( )12 hhnQC = &&

    which upon combination with Equation 3.89 gives

    ( )42 hhnQC = &&

    From the NIST website:

    =molkJ 295.392h (sat. vapor at 0.12 MPa)

    =molkJ 181.244h (sat. liquid at 0.7 MPa)

    Therefore,

    [ ]kW557.7molkJ181.24

    molkJ 295.39

    smol 5.0 =

    =C Q&

    (b) The power input to the compressor can be calculated with Equation 3.86:

    ( )23 hhnW C = &&

    Equation 3.87 states

    32 ss =

  • 8/13/2019 felix termodinamica quimica ch03

    88/104

    88

    From the NIST website:

    ( )==K mol

    J 89.177MPa0.12atvaporsat.23 ss

    Now, state 3 is constrained.

    =molkJ 031.433h

    =

    K molJ 89.177MPa,7.0 3s

    Therefore,

    [ ]kW87.1molkJ 295.39

    molkJ 031.43

    smol 5.0 =

    =C W &

    (c)Equation 3.90 states:

    05.4

    molkJ 295.39

    molkJ 031.43

    molkJ 181.24

    molkJ 295.39

    2342

    2312 =

    =

    =

    =

    hhhh

    hhhh

    COP

  • 8/13/2019 felix termodinamica quimica ch03

    89/104

    89

    3.41Using the information in the problem statement and our knowledge of ideal vapor compressioncycles, the following table can be created

    State Thermodynamic

    State

    P (MPa)

    1 Sat. Mixture 0.122 Sat. Vapor 0.123 Superheated Vapor 0.74 Sat. Liquid 0.7

    Note: Refer to Figure 3.9 to review labeling convention.

    From Equation 3.90

    23

    12

    hhhh

    W Q

    COPC

    C

    == &

    &

    The enthalpy of state 2 can be found directly from the NIST website, but the enthalpies of state1 and 3 require the use of additional information.

    =molkJ 295.392h (sat. vapor at 0.12 MPa)

    For the process between state 2 and state 3,

    32 ss =

    From the NIST website:

    ==K mol

    J 89.17723 ss (sat. vapor at 0.12 MPa)

    Now, state 3 is constrained.

    =molkJ 031.433h

    =

    K molJ 89.177MPa,7.0 3s

    The process between state 4 and state 1 is also isentropic.

    ==K mol

    J 09.11541 ss (sat. liquid at 0.7 MPa)

    The quality of the R134a can be calculated as follows

  • 8/13/2019 felix termodinamica quimica ch03

    90/104

    90

    ( ) vl xss xs 111 1 +=

    where

    =K mol

    J 649.901ls (sat. R134a(l) at 0.12 MPa)

    =K mol

    J 89.1771vs (sat. R134a(v) at 0.12 MPa)

    Therefore,

    ( )

    +

    =

    K molJ 89.177

    K molJ 649.901

    K molJ 09.115 x x

    280.0= x Using the quality of R134a, the enthalpy of state 1 can be calculated as follows

    ( ) vl xhh xh 111 1 +=

    From the NIST website:

    =molkJ 412.171lh (sat. R134a(l) at 0.12 MPa)

    =molkJ 295.391vh (sat. R134a(v) at 0.12 MPa)

    Therefore,

    ( )

    +

    =

    molkJ 295.3928.0

    molkJ 412.1728.011h

    =molkJ 74.201h

    Now, everything needed to calculate COP is available. Using Equation 3.90,

    97.4

    molkJ 295.39

    molkJ 031.43

    molkJ 74.20

    molkJ 295.39

    2312 =

    =

    =

    hhhh

    COP

  • 8/13/2019 felix termodinamica quimica ch03

    91/104

    91

    Is this modification practical? No. An isentropic turbine adds significant level of complexity to the cycle. Turbines areexpensive and wear over time. Furthermore, the real turbine added to the cycle will not be 100efficient, so the COP will not increase as much. The cost of the turbine is not justified by theincrease in COP.

  • 8/13/2019 felix termodinamica quimica ch03

    92/104

  • 8/13/2019 felix termodinamica quimica ch03

    93/104

    93

    (b)Performing an energy balance we find

    ( )56 hhnQ cC = &&

    We can also use the following relationship

    ==mol

    J 2109985 hh

    From the NIST website:

    =mol

    J392956h (sat. R134 vapor at 0.12 MPa)

    Therefore,

    [ ]kW48.7mol

    J 21099mol

    J39295s

    mol 411.0 =

    =C Q&

    (c)The power input is calculated as follows:

    hC cC totalC W W W ,,, &&& +=

    where

    ( )67, hhnW ccC = && ( )23, hhnW hhC = &&

    We have all of the required enthalpies excepth3. State 3 is constrained because

    ==K mol

    J95.17523 ss (sat. R134a vapor at 0.35 MPa)

    MPa7.03 =P

    From the NIST website:

    =mol

    J424283h (R134a vapor at 0.7 MPa with= K molJ95.175s )

    Now compute the power for each unit:

  • 8/13/2019 felix termodinamica quimica ch03

    94/104

    94

    [ ] [ ]

    +

    =

    molJ 40967

    molJ 42428mol/s5.0

    molJ 39295

    molJ 41510mol/s411.0,totalC W &

    [ ]kW64.1, =totalC W &

    (d)The coefficient of performance is calculated using the following equation:

    56.4kW64.1kW48.7

    ,===

    totalC

    C W

    QCOP &

    &

    (e)The COP for the cascade is 4.56, while the COP is 4.05 in Problem 3.40. The cascade systemCOP is 12.6% greater.

  • 8/13/2019 felix termodinamica quimica ch03

    95/104

    95

    3.43One design follows; your design may differ:

    In order to cool a system to -5 C, the temperature of the fluid must be colder than -5 C so thatheat transfer will occur. We arbitrarily specify that the working fluid evaporates at -10 C.

    Similarly, in order to eject heat to the 20 C reservoir, the fluid must condense at a temperaturegreater than 20 C. Arbitrarily, we choose 25 C. One possible refrigeration cycle is presented below.

    evaporator

    compressor

    condenser

    1

    4

    3

    2

    T

    s

    3

    21

    4

    valve

    refrigeratedunit at -5 C

    20 Creservoir

    W c

    Q H

    Q H

    QC

    QC

    A number of fluids will work sufficiently for this system, but the design process will beillustrated using R134a. For states 3 and 4, the pressure is constant, and for states 1 and 2, the pressure is constant at a different value. From the NIST website, we find

    MPa201.021 == PP (T sat = -10 C)MPa665.043 == PP (T sat = 25 C)

    (Note: The temperatures of each state are not constant. The listed saturation temperatureare the temperatures at which the fluid evaporates and condenses.)

    Now that the pressures are known, we can compute the required flow rate required in order to provide 20 kW of cooling.

    ( )12 hhnQC = &&

    We can geth2 from the thermodynamic tables for saturated R134a. In order to findh1 , we canuse the following relationship:

    41 hh =

    From the NIST website:

    =mol

    J 400642h (sat. R134 vapor at 0.201 MPa)

  • 8/13/2019 felix termodinamica quimica ch03

    96/104

    96

    =mol

    J 239314h (sat. R134 liquid at 0.665 MPa)

    Now, calculate the required flow rate of R134a.

    ( )[ ]

    =

    =

    molJ 23931

    molJ 40064

    J/s000,2012 hh

    Qn C

    &&

    =s

    mol24.1n&

  • 8/13/2019 felix termodinamica quimica ch03

    97/104

    97

    3.44The schematic of the process and the correspondingTs is presented below

    evaporator

    compressor

    condenser

    1

    63

    2T

    s

    2

    1

    4

    valve

    high Treservoir

    valve

    Fridge

    Freezer evaporator

    54

    T = -15 oC

    T = 5 oC

    3

    5

    6

    5 oC

    -15 oC

    QC,R

    QC,F Q H

    Q H

    QC,F

    QC,R

    W c

    W c

    A number of refrigerants will work for this system, but the design process will be illustrated forR134a only. To define each state, we need to thermodynamically constrain each state. Youshould note that the problem doesnt state what temperature the fluid condenses at. Therefore,we can assume the temperature is 25 C. By using information in the Ts diagram and from the NIST website, we find

    MPa34966.021 == PP (T 1=T 2=T sat = 5 C)MPa16394.043 == PP (T 3=T 4=T sat = -15 C)MPa66538.065 == PP (T 6 = 25 C)

    Before solving for additional pressures and temperatures, we will list the known temperatures

    and pressures.State Temperature (C) Pressure (MPa)

    1 5 0.243342 5 0.243343 -15 0.163944 -15 0.163945 25 0.665386 25 0.66538

    States 4, 5, and 6 are completely constrained as confirmed by Gibbs phase rule. Now, we needto find the liquid and vapor compositions of states 1, 2, and 3 to completely constrain the statesSince the heat duties are equal for the refrigerator and the freezer, we have the followingrelationship:

    RC F C QQ ,, && =

  • 8/13/2019 felix termodinamica quimica ch03

    98/104

    98

    which upon performing an energy balance becomes

    1234 hhhh =

    Energy balances around the valves provide:

    61 hh = 32 hh =

    Substituting these relationships into the expression that equates the heat loads results in

    264

    32hh

    hh +==

    From the NIST website:

    =mol

    J 397554h

    =mol

    J 239316h

    Now constrain states 1, 2, and 3 by determining the enthalpies:

    ==mol

    J 2393161 hh

    =+

    ==mol

    J 318432

    molJ 23931

    molJ 39755

    32 hh

    Technically, the states are now all constrained, but we would like also like to know the vapor anliquid compositions of each state. The compositions of states 4, 5, and 6 are already known; wcan calculate the compositions of states 1, 2, and 3 as follows:

    ( ) sat vsat

    l h xh xh ,11,111 1 +=

    ( ) sat

    vsat

    l h xh xh ,22,222 1 += ( ) sat v

    sat l h xh xh ,33,333 1 +=

    From the NIST website,

  • 8/13/2019 felix termodinamica quimica ch03

    99/104

    99

    =mol

    J 21095,1sat

    lh = molJ 40965,1

    sat vh

    =mol

    J 21095,2sat

    lh = molJ 40965,2

    sat vh

    = molJ 18380,3sat lh = molJ 39755,3

    sat vh

    Now, we can solve the composition of vapor for each state:

    143.01 = x 541.02 = x 630.03 = x

    The following table presents a summary of our results:

    State Temperature(C)Pressure

    (MPa) PhasesLiquid

    CompositionVapor

    Composition

    1 5 0.24334 Saturated Liquidand Vapor 0.857 0.143

    2 5 0.24334 Saturated Liquidand Vapor 0.459 0.541

    3 -15 0.16394 Saturated Liquidand Vapor 0.37 0.630

    4 -15 0.16394 Saturated Vapor 0 1

    5 25 0.66538 SuperheatedVapor 0 1

    6 25 0.66538 Saturated Liquid 1 0

  • 8/13/2019 felix termodinamica quimica ch03

    100/104

    100

    3.45In this case, the working material is a solid. The four states of the magnetic material are shownof thesT diagram below. Note the axis are shifted from the usual manner.

    1

    2

    3

    4

    QC Q H

    (a) The heat expelled by the cold reservoir can be approximated by:

    ( )12 ssT q C C

    whereT is the average temperature between states 1 and 2, which is approximately 1 K. Takinvalues ofS/R from thesT diagram, we get:

    ( )= K molJ0.53.19.1 RqC

    (b) Similarly, the heat absorbed by the hot reservoir can be approximated by:

    ( ) ( )==K mol

    J519.12.18.844 RssT q H H

    (b) The coefficient of performance is given by:

    11.0,

    =

    ==C H

    C

    totalC

    C

    qqq

    wq

    COP

    (d) The value of COP is much lower than a typical refrigeration process (COP= 4-6); in generarefrigeration processes at these low temperatures are much less efficient.

    F. Work is supplied to magnetize the material and to spin the wheel.

  • 8/13/2019 felix termodinamica quimica ch03

    101/104

    101

    3.46 When the polymer is unstretched it is in a more entangled state. When stretched the polymerchains tend to align. The alignment decreases the spatial configurations the polymer can have,and therefore, reduces that component of entropy. If the process is adiabatic, the entropy of thesystem cannot decrease. Consequently, its thermal entropy must increase. The only way this c

    be accomplished is by increased temperature.

  • 8/13/2019 felix termodinamica quimica ch03

    102/104

    102

    3.47Assume the temperature is 298 K. The following data was taken from Table A.3.2:

    Species f h (kJ/mol) f g (kJ/mol)Cu2O (s) -170.71 -147.88O2 (g) 0 0

    CuO (s) -156.06 -128.29

    The data listed above were used to create the next table using

    T

    ghs f f f

    =

    Species f s (kJ/mol K)

    Cu2O (s) -0.0766O2 (g) 0CuO (s) -0.0932

    Now the change in entropy of the reaction can be calculated in a method analogous to Equation2.72

    ( )

    +

    == K mol

    kJ 0.0932-4K mol

    kJ 0.0766-2i f irxn

    svs

    =mol

    kJ 22.0rxns

    Does this violate the second law of thermodynamics? This problem shows that the entropychange of the system is negative, but nothing has been said about the entropy change of theuniverse. We must look at the change in entropy of the surroundings to determine if the secondlaw is violated. By looking at the enthalpies, we see that the reaction is exothermic, whichmeans that heat is transferred from the system to the surroundings. Therefore, the entropy of thsurroundings will increase during this reaction.

  • 8/13/2019 felix termodinamica quimica ch03

    103/104

    103

    3.48The temperature and pressure terms in the equation for entropy do not contribute anything to thentropy change because they are constant. Therefore, the only remaining entropy contribution,the randomness of the atomic arrangements, must be considered. The randomness does notincrease when CdTe forms. In pure crystals of Cd and Te, the location of each atom is known

    because the crystal lattice constrains the atomic locations. In CdTe, the crystal lattice stilldefines the location of each atom, so the randomness has not increased. Therefore, the change entropy is zero.

    3.49This argument is not scientifically sound. Morris is arguing that since evolution results in moreorder, the second law of thermodynamics is violated, so evolution must be impossible. Howevthe flaw in this argument is caused by ignoring the entropy change of the entire universe. Thesecond law states that the entropyof the universe will remain constant or increase for any process. Morris argument was based on the entropy of the system undergoing evolution notthe entropy of the entire universe. A system can decrease in entropy if the entropy of the

    surroundings increases by at least that much.

  • 8/13/2019 felix termodinamica quimica ch03

    104/104

    3.50Entropy is related to the number of configurations that a state can have. The greater the numbeof configurations, the more probable the state is and the greater the entropy. We canqualitatively relate this concept to the possible hands in a game of poker, but it is moreinteresting to quantify the results using some basic concepts of probability.

    We consider a hand of poker containing 5 cards randomly draw from a deck of 52 cards.. Therare a finite number of permutations in which we can arrange a 52 card deck in 5 cards. For thefirst card in the hand, we select from 52 cards, the second card can be any other card so we selefrom 51 cards, the third card has 50 cards, and so on. Thus the number of permutations of 5cards is:

    P =5251504948 =311,875,200

    However, we do not care the order in which the cards are dealt, so we must divide this number by the number of ways we can come up with the same hand. We do this math in a similar way.

    For a given hand there are five cards we can pick first, four we can pick second, . So thenumber of ways we can make the same hand is:

    N =54321=120

    The number of unique configurations can be found by dividingP by N . Thus, the cards candisplay

    C = P N

    =2,598,960

    or 2,598,960 unique configurations. To find the entropy of a given hand, we need to find ouhow many of these unique configurations belong to the hand

    Consider a four of a kind. There are 13 different possible ranks of four of a kind, one for eachnumber A, 2, 3, 4, 5, 6, 7,8, 9, 10, J, Q, K. The fifth card in the hand could be any of the other48 cards. Therefore, the number of combinations of four of a kind is: 13 x 48 = 624